Difference between revisions of "2024 USAJMO Problems/Problem 3"
(→Solution 1) |
|||
Line 6: | Line 6: | ||
== Solution 1 == | == Solution 1 == | ||
+ | Lemma <math>1</math>: | ||
+ | |||
+ | Given a prime <math>p</math>, a positive integer <math>k</math>, and an even <math>m</math> such that <math>p^k|a(m)</math>, we must have that <math>p^{k+1}|a(m+2)</math>. | ||
+ | |||
+ | Proof of Lemma <math>1</math>: | ||
+ | |||
+ | <math>a(m+1)\equiv (a(m))^{m+1}-1\equiv -1 \mod p^{k+1}</math> | ||
+ | |||
+ | Therefore, <math>a(m+2)\equiv (a(m+1))^{m+2}-1\equiv (-1)^{m+2}-1\equiv 0 \mod p^{k+1}</math> | ||
==See Also== | ==See Also== | ||
{{USAJMO newbox|year=2024|num-b=2|num-a=4}} | {{USAJMO newbox|year=2024|num-b=2|num-a=4}} | ||
{{MAA Notice}} | {{MAA Notice}} |
Revision as of 08:31, 28 March 2024
Contents
Problem
Let be the sequence defined by and for each integer . Suppose that is prime and is a positive integer. Prove that some term of the sequence is divisible by .
Solution 1
Lemma :
Given a prime , a positive integer , and an even such that , we must have that .
Proof of Lemma :
Therefore,
See Also
2024 USAJMO (Problems • Resources) | ||
Preceded by Problem 2 |
Followed by Problem 4 | |
1 • 2 • 3 • 4 • 5 • 6 | ||
All USAJMO Problems and Solutions |
The problems on this page are copyrighted by the Mathematical Association of America's American Mathematics Competitions.